A curious prime counting approximation or just data overfitting?Is Li(x) the best possible approximation to...



A curious prime counting approximation or just data overfitting?


Is Li(x) the best possible approximation to the prime-counting function?Estimate on the prime-counting function $psi(x)$.Asymptotic bounds on $pi^{-1}(x)$ (inverse prime counting function)Prime-Counting Functiona question for the prime counting functionThe shortest interval for which the prime number theorem holdsParity of the Prime Counting Functionprime counting function pi boundsnth prime better approximationPrime counting. Meissel, Lehmer: is there a general formula?













3












$begingroup$


I am not sure, if this is a research problem. If not I will move this question to ME:
Let $Omega(n) = sum_{p|n} v_p(n)$, which we might view as a random variable.
Let $E_n = frac{1}{n} sum_{k=1}^nOmega(k)$ be the expected value and $V_n=frac{1}{n} sum_{k=1}^n(E_n-Omega(k))^2$ be the variance.
Then
$$pi(n) approx frac{ngamma(frac{V_n}{E_n},1.4854177cdot frac{V_n}{E_n^2})}{Gamma(frac{V_n}{E_n})}$$
where $Gamma=$ Gamma function, $gamma=$ lower incomplete gamma function.



Background:
I was trying to fit the gamma distribution to the random variable $Omega(k)$ ,$1 le k le n$. The value $1.4854177$ is fitted to some data.
My question is, if there is any heuristic why this approximation should be good, if at all, or if this is just an overfitting problem?



Below you can find some sage code which implements this:



def Omega(n):
return sum([valuation(n,p) for p in prime_divisors(n)])

means = []
variances = []
xxs = []
omegas = [Omega(k) for k in range(1,10^4)]
for nn in range(10^4,10^4+3*10^3+1):
n = nn
omegas.append(Omega(n))
print "---"
m = mean(omegas[1:-1])
v = variance(omegas[1:-1])
shape,scale = m^2/v,v/m
xx = find_root(lambda xx : n*(lower_gamma(shape,xx*1/scale)/gamma(shape) ).N()-prime_pi(n),1,2)
xx = 1.4854177706344873
approxPrimePi2 = n*(lower_gamma(shape,xx*1/scale)/gamma(shape) ).N()
primepi = prime_pi(n)
print primepi, approxPrimePi2,shape.N(),scale.N(),xx
print "---"
print "err2 = %s" % (abs(primepi-approxPrimePi2)/primepi)
xxs.append(xx)
means.append(m.N())
variances.append(v.N())









share|cite|improve this question











$endgroup$












  • $begingroup$
    Does $1.485...$ correspond to some known constant?
    $endgroup$
    – Sylvain JULIEN
    7 hours ago












  • $begingroup$
    I am not sure. Maybe.
    $endgroup$
    – orgesleka
    7 hours ago
















3












$begingroup$


I am not sure, if this is a research problem. If not I will move this question to ME:
Let $Omega(n) = sum_{p|n} v_p(n)$, which we might view as a random variable.
Let $E_n = frac{1}{n} sum_{k=1}^nOmega(k)$ be the expected value and $V_n=frac{1}{n} sum_{k=1}^n(E_n-Omega(k))^2$ be the variance.
Then
$$pi(n) approx frac{ngamma(frac{V_n}{E_n},1.4854177cdot frac{V_n}{E_n^2})}{Gamma(frac{V_n}{E_n})}$$
where $Gamma=$ Gamma function, $gamma=$ lower incomplete gamma function.



Background:
I was trying to fit the gamma distribution to the random variable $Omega(k)$ ,$1 le k le n$. The value $1.4854177$ is fitted to some data.
My question is, if there is any heuristic why this approximation should be good, if at all, or if this is just an overfitting problem?



Below you can find some sage code which implements this:



def Omega(n):
return sum([valuation(n,p) for p in prime_divisors(n)])

means = []
variances = []
xxs = []
omegas = [Omega(k) for k in range(1,10^4)]
for nn in range(10^4,10^4+3*10^3+1):
n = nn
omegas.append(Omega(n))
print "---"
m = mean(omegas[1:-1])
v = variance(omegas[1:-1])
shape,scale = m^2/v,v/m
xx = find_root(lambda xx : n*(lower_gamma(shape,xx*1/scale)/gamma(shape) ).N()-prime_pi(n),1,2)
xx = 1.4854177706344873
approxPrimePi2 = n*(lower_gamma(shape,xx*1/scale)/gamma(shape) ).N()
primepi = prime_pi(n)
print primepi, approxPrimePi2,shape.N(),scale.N(),xx
print "---"
print "err2 = %s" % (abs(primepi-approxPrimePi2)/primepi)
xxs.append(xx)
means.append(m.N())
variances.append(v.N())









share|cite|improve this question











$endgroup$












  • $begingroup$
    Does $1.485...$ correspond to some known constant?
    $endgroup$
    – Sylvain JULIEN
    7 hours ago












  • $begingroup$
    I am not sure. Maybe.
    $endgroup$
    – orgesleka
    7 hours ago














3












3








3





$begingroup$


I am not sure, if this is a research problem. If not I will move this question to ME:
Let $Omega(n) = sum_{p|n} v_p(n)$, which we might view as a random variable.
Let $E_n = frac{1}{n} sum_{k=1}^nOmega(k)$ be the expected value and $V_n=frac{1}{n} sum_{k=1}^n(E_n-Omega(k))^2$ be the variance.
Then
$$pi(n) approx frac{ngamma(frac{V_n}{E_n},1.4854177cdot frac{V_n}{E_n^2})}{Gamma(frac{V_n}{E_n})}$$
where $Gamma=$ Gamma function, $gamma=$ lower incomplete gamma function.



Background:
I was trying to fit the gamma distribution to the random variable $Omega(k)$ ,$1 le k le n$. The value $1.4854177$ is fitted to some data.
My question is, if there is any heuristic why this approximation should be good, if at all, or if this is just an overfitting problem?



Below you can find some sage code which implements this:



def Omega(n):
return sum([valuation(n,p) for p in prime_divisors(n)])

means = []
variances = []
xxs = []
omegas = [Omega(k) for k in range(1,10^4)]
for nn in range(10^4,10^4+3*10^3+1):
n = nn
omegas.append(Omega(n))
print "---"
m = mean(omegas[1:-1])
v = variance(omegas[1:-1])
shape,scale = m^2/v,v/m
xx = find_root(lambda xx : n*(lower_gamma(shape,xx*1/scale)/gamma(shape) ).N()-prime_pi(n),1,2)
xx = 1.4854177706344873
approxPrimePi2 = n*(lower_gamma(shape,xx*1/scale)/gamma(shape) ).N()
primepi = prime_pi(n)
print primepi, approxPrimePi2,shape.N(),scale.N(),xx
print "---"
print "err2 = %s" % (abs(primepi-approxPrimePi2)/primepi)
xxs.append(xx)
means.append(m.N())
variances.append(v.N())









share|cite|improve this question











$endgroup$




I am not sure, if this is a research problem. If not I will move this question to ME:
Let $Omega(n) = sum_{p|n} v_p(n)$, which we might view as a random variable.
Let $E_n = frac{1}{n} sum_{k=1}^nOmega(k)$ be the expected value and $V_n=frac{1}{n} sum_{k=1}^n(E_n-Omega(k))^2$ be the variance.
Then
$$pi(n) approx frac{ngamma(frac{V_n}{E_n},1.4854177cdot frac{V_n}{E_n^2})}{Gamma(frac{V_n}{E_n})}$$
where $Gamma=$ Gamma function, $gamma=$ lower incomplete gamma function.



Background:
I was trying to fit the gamma distribution to the random variable $Omega(k)$ ,$1 le k le n$. The value $1.4854177$ is fitted to some data.
My question is, if there is any heuristic why this approximation should be good, if at all, or if this is just an overfitting problem?



Below you can find some sage code which implements this:



def Omega(n):
return sum([valuation(n,p) for p in prime_divisors(n)])

means = []
variances = []
xxs = []
omegas = [Omega(k) for k in range(1,10^4)]
for nn in range(10^4,10^4+3*10^3+1):
n = nn
omegas.append(Omega(n))
print "---"
m = mean(omegas[1:-1])
v = variance(omegas[1:-1])
shape,scale = m^2/v,v/m
xx = find_root(lambda xx : n*(lower_gamma(shape,xx*1/scale)/gamma(shape) ).N()-prime_pi(n),1,2)
xx = 1.4854177706344873
approxPrimePi2 = n*(lower_gamma(shape,xx*1/scale)/gamma(shape) ).N()
primepi = prime_pi(n)
print primepi, approxPrimePi2,shape.N(),scale.N(),xx
print "---"
print "err2 = %s" % (abs(primepi-approxPrimePi2)/primepi)
xxs.append(xx)
means.append(m.N())
variances.append(v.N())






nt.number-theory analytic-number-theory prime-numbers prime-number-theorem






share|cite|improve this question















share|cite|improve this question













share|cite|improve this question




share|cite|improve this question








edited 7 hours ago









GH from MO

60.8k5153232




60.8k5153232










asked 8 hours ago









orgeslekaorgesleka

724417




724417












  • $begingroup$
    Does $1.485...$ correspond to some known constant?
    $endgroup$
    – Sylvain JULIEN
    7 hours ago












  • $begingroup$
    I am not sure. Maybe.
    $endgroup$
    – orgesleka
    7 hours ago


















  • $begingroup$
    Does $1.485...$ correspond to some known constant?
    $endgroup$
    – Sylvain JULIEN
    7 hours ago












  • $begingroup$
    I am not sure. Maybe.
    $endgroup$
    – orgesleka
    7 hours ago
















$begingroup$
Does $1.485...$ correspond to some known constant?
$endgroup$
– Sylvain JULIEN
7 hours ago






$begingroup$
Does $1.485...$ correspond to some known constant?
$endgroup$
– Sylvain JULIEN
7 hours ago














$begingroup$
I am not sure. Maybe.
$endgroup$
– orgesleka
7 hours ago




$begingroup$
I am not sure. Maybe.
$endgroup$
– orgesleka
7 hours ago










1 Answer
1






active

oldest

votes


















8












$begingroup$

Your heuristic approximation is not correct. It was proved by Turán (1934) that
$E_n$ and $V_n$ are both asymptotically $loglog n$. As a result, the RHS of your display is
$$nfrac{gammaleft(1+o(1),frac{1.4854177+o(1)}{loglog n}right)}{Gammabigl(1+o(1)bigr)}=nfrac{1.4854177+o(1)}{loglog n}.$$
On the other hand, $pi(n)$ is asymptotically $n/log n$ by the prime number theorem.






share|cite|improve this answer











$endgroup$









  • 1




    $begingroup$
    thanks. i thought that there must be some mistake
    $endgroup$
    – orgesleka
    7 hours ago












Your Answer








StackExchange.ready(function() {
var channelOptions = {
tags: "".split(" "),
id: "504"
};
initTagRenderer("".split(" "), "".split(" "), channelOptions);

StackExchange.using("externalEditor", function() {
// Have to fire editor after snippets, if snippets enabled
if (StackExchange.settings.snippets.snippetsEnabled) {
StackExchange.using("snippets", function() {
createEditor();
});
}
else {
createEditor();
}
});

function createEditor() {
StackExchange.prepareEditor({
heartbeatType: 'answer',
autoActivateHeartbeat: false,
convertImagesToLinks: true,
noModals: true,
showLowRepImageUploadWarning: true,
reputationToPostImages: 10,
bindNavPrevention: true,
postfix: "",
imageUploader: {
brandingHtml: "Powered by u003ca class="icon-imgur-white" href="https://imgur.com/"u003eu003c/au003e",
contentPolicyHtml: "User contributions licensed under u003ca href="https://creativecommons.org/licenses/by-sa/3.0/"u003ecc by-sa 3.0 with attribution requiredu003c/au003e u003ca href="https://stackoverflow.com/legal/content-policy"u003e(content policy)u003c/au003e",
allowUrls: true
},
noCode: true, onDemand: true,
discardSelector: ".discard-answer"
,immediatelyShowMarkdownHelp:true
});


}
});














draft saved

draft discarded


















StackExchange.ready(
function () {
StackExchange.openid.initPostLogin('.new-post-login', 'https%3a%2f%2fmathoverflow.net%2fquestions%2f333258%2fa-curious-prime-counting-approximation-or-just-data-overfitting%23new-answer', 'question_page');
}
);

Post as a guest















Required, but never shown

























1 Answer
1






active

oldest

votes








1 Answer
1






active

oldest

votes









active

oldest

votes






active

oldest

votes









8












$begingroup$

Your heuristic approximation is not correct. It was proved by Turán (1934) that
$E_n$ and $V_n$ are both asymptotically $loglog n$. As a result, the RHS of your display is
$$nfrac{gammaleft(1+o(1),frac{1.4854177+o(1)}{loglog n}right)}{Gammabigl(1+o(1)bigr)}=nfrac{1.4854177+o(1)}{loglog n}.$$
On the other hand, $pi(n)$ is asymptotically $n/log n$ by the prime number theorem.






share|cite|improve this answer











$endgroup$









  • 1




    $begingroup$
    thanks. i thought that there must be some mistake
    $endgroup$
    – orgesleka
    7 hours ago
















8












$begingroup$

Your heuristic approximation is not correct. It was proved by Turán (1934) that
$E_n$ and $V_n$ are both asymptotically $loglog n$. As a result, the RHS of your display is
$$nfrac{gammaleft(1+o(1),frac{1.4854177+o(1)}{loglog n}right)}{Gammabigl(1+o(1)bigr)}=nfrac{1.4854177+o(1)}{loglog n}.$$
On the other hand, $pi(n)$ is asymptotically $n/log n$ by the prime number theorem.






share|cite|improve this answer











$endgroup$









  • 1




    $begingroup$
    thanks. i thought that there must be some mistake
    $endgroup$
    – orgesleka
    7 hours ago














8












8








8





$begingroup$

Your heuristic approximation is not correct. It was proved by Turán (1934) that
$E_n$ and $V_n$ are both asymptotically $loglog n$. As a result, the RHS of your display is
$$nfrac{gammaleft(1+o(1),frac{1.4854177+o(1)}{loglog n}right)}{Gammabigl(1+o(1)bigr)}=nfrac{1.4854177+o(1)}{loglog n}.$$
On the other hand, $pi(n)$ is asymptotically $n/log n$ by the prime number theorem.






share|cite|improve this answer











$endgroup$



Your heuristic approximation is not correct. It was proved by Turán (1934) that
$E_n$ and $V_n$ are both asymptotically $loglog n$. As a result, the RHS of your display is
$$nfrac{gammaleft(1+o(1),frac{1.4854177+o(1)}{loglog n}right)}{Gammabigl(1+o(1)bigr)}=nfrac{1.4854177+o(1)}{loglog n}.$$
On the other hand, $pi(n)$ is asymptotically $n/log n$ by the prime number theorem.







share|cite|improve this answer














share|cite|improve this answer



share|cite|improve this answer








edited 7 hours ago

























answered 7 hours ago









GH from MOGH from MO

60.8k5153232




60.8k5153232








  • 1




    $begingroup$
    thanks. i thought that there must be some mistake
    $endgroup$
    – orgesleka
    7 hours ago














  • 1




    $begingroup$
    thanks. i thought that there must be some mistake
    $endgroup$
    – orgesleka
    7 hours ago








1




1




$begingroup$
thanks. i thought that there must be some mistake
$endgroup$
– orgesleka
7 hours ago




$begingroup$
thanks. i thought that there must be some mistake
$endgroup$
– orgesleka
7 hours ago


















draft saved

draft discarded




















































Thanks for contributing an answer to MathOverflow!


  • Please be sure to answer the question. Provide details and share your research!

But avoid



  • Asking for help, clarification, or responding to other answers.

  • Making statements based on opinion; back them up with references or personal experience.


Use MathJax to format equations. MathJax reference.


To learn more, see our tips on writing great answers.




draft saved


draft discarded














StackExchange.ready(
function () {
StackExchange.openid.initPostLogin('.new-post-login', 'https%3a%2f%2fmathoverflow.net%2fquestions%2f333258%2fa-curious-prime-counting-approximation-or-just-data-overfitting%23new-answer', 'question_page');
}
);

Post as a guest















Required, but never shown





















































Required, but never shown














Required, but never shown












Required, but never shown







Required, but never shown

































Required, but never shown














Required, but never shown












Required, but never shown







Required, but never shown







Popular posts from this blog

Taj Mahal Inhaltsverzeichnis Aufbau | Geschichte | 350-Jahr-Feier | Heutige Bedeutung | Siehe auch |...

Baia Sprie Cuprins Etimologie | Istorie | Demografie | Politică și administrație | Arii naturale...

Nicolae Petrescu-Găină Cuprins Biografie | Opera | In memoriam | Varia | Controverse, incertitudini...